Buy BOOKS at Discounted Price

Real Numbers

Class 10th Mathematics Rajasthan Board Solution
Exercise 2.1
  1. Show that the square of an odd positive integer is of the form 8q + 1, where q…
  2. By Euclid Division Lemma show that the cube of any positive integral number is…
  3. Show that any positive odd integral number can be expressed as 6q + 1 or 6q + 3…
  4. 210, 55 Find the HCF (Highest Common Factor) of the following pairs of number…
  5. 420, 130 Find the HCF (Highest Common Factor) of the following pairs of number…
  6. 75, 243 Find the HCF (Highest Common Factor) of the following pairs of number…
  7. 135, 225 Find the HCF (Highest Common Factor) of the following pairs of number…
  8. 196, 38220 Find the HCF (Highest Common Factor) of the following pairs of…
  9. 867, 255 Find the HCF (Highest Common Factor) of the following pairs of number…
  10. If the highest common factor of numbers 408 and 1032 is expressed in the form of…
Exercise 2.2
  1. 468 Express the following numbers as a product of its prime factors:…
  2. 945 Express the following numbers as a product of its prime factors:…
  3. 140 Express the following numbers as a product of its prime factors:…
  4. 3825 Express the following numbers as a product of its prime factors:…
  5. 20570 Express the following numbers as a product of its prime factors:…
  6. 96 and 404 Find the HCF and LCM of the following pairs of integers and verify…
  7. 336 and 54 Find the HCF and LCM of the following pairs of integers and verify…
  8. 90 and 144 Find the HCF and LCM of the following pairs of integers and verify…
  9. 12, 15 and 21 Find the HCF and LCM of the following integers by applying the…
  10. 24, 15 and 36 Find the HCF and LCM of the following integers by applying the…
  11. 17, 23 and 29 Find the HCF and LCM of the following integers by applying the…
  12. 6, 72 and 120 Find the HCF and LCM of the following integers by applying the…
  13. 40, 36 and 126 Find the HCF and LCM of the following integers by applying the…
  14. 8, 9 and 25 Find the HCF and LCM of the following integers by applying the…
  15. Raman takes 18 minutes to drive one round of the field on circular path of a…
  16. In a seminar, the number of participants in Hindi, English and Mathematics are…
Exercise 2.3
  1. Prove that 5 - root {3} is an irrational number.
  2. {1}/{ root {2} } Prove that the following numbers are irrational numbers:…
  3. 6 + root {2} Prove that the following numbers are irrational numbers:…
  4. 3 root {2} Prove that the following numbers are irrational numbers:…
  5. If p and q are co-prime positive integers then prove that root {p} + sqrt{q}…
Exercise 2.4
  1. Without actually performing the long division, state whether the following…
  2. Write the decimal expansions of the following rational numbers and decide…
  3. For the decimal expansions given below decide whether this number is a rational…
Miscellaneous Exercise 2
  1. The sum of the powers of the prime factors of 196 is:
  2. If two numbers are written in the form of m = pq3 and n = p3q2 then find the HCF…
  3. The HCF (Highest Common Factor) of 95 and 152 is :
  4. The product of two numbers is 1080. Their HCF is 30. Then their LCM is:…
  5. The decimal expansion of the number {441}/{ 2^{2} x 5^{7} times 7^{2} }…
  6. After how many decimal places with the rational number {43}/{ 2^{2} x 5^{3}…
  7. The least number by which multiplying to root {27} a rational number is…
  8. If for two rational numbers HCF = LCM, then the number must use:
  9. If LCM of a and 18 is 36 and HCF of a and 18 is 2, then the value of a will be:…
  10. If n is a rational number, then units digit in 6n – 5n is:
  11. If {p}/{q} ( q not equal 0 ) is a rational number, then what will be the…
  12. Simplifying state whether the number { 2 root {45}+3 sqrt{20} }/{ 2 sqrt{5}…
  13. Show that any positive odd integer is of the form 4q + 1 or 4q + 3. Where q is…
  14. Prove that the product of two executive positive integers is divisible by 2.…
  15. Find the greatest number by which on dividing 2056 and 967 the remainder are…
  16. Explain why 7 × 11 × 13 + 13 and 7 × 6 × 5 × 4 × 3 × 2 × 1 + 5 are composite…
  17. If the HCF of two numbers 306 and 657 is 9 then find their LCM.
  18. A rectangular verandah is 18 m 72 cm long and 13 m 20 cm wide. Square tiles of…
  19. 5 root {2} Prove that the following numbers are irrational numbers:…
  20. {2}/{ root {7} } Prove that the following numbers are irrational numbers:…
  21. {3}/{ 2 root {5} } Prove that the following numbers are irrational numbers:…
  22. 2 + root {2} Prove that the following numbers are irrational numbers:…
  23. What can you say about the prime factorization of the denominators of the…

Exercise 2.1
Question 1.

Show that the square of an odd positive integer is of the form 8q + 1, where q is a positive integer.


Answer:

Let a be any positive integer.

Then a = 8m + 1 where m is some integer


a = 8m + 1


⟹ a2 = (8m + 1)2


= 64m2 + 16m + 1


= 8q + 1


Where q = 8m2 + 2m


Hence square of an odd integer is of the form 8q + 1 where q is a positive integer.



Question 2.

By Euclid Division Lemma show that the cube of any positive integral number is of the form 9q or 9q + 1 or 9q + 8, where q is an integral number.


Answer:

According to the Euclid’s Division Lemma,

Let a be any positive integer and b = 9


Then by Euclid’s Lemma we have,


a = 9q + r


Where r = 0, 1, 2, 3, 4, 5, 6, 7, 8 [since 0 ≤ r ≤ b] and here value of b is 9]


So all the possible forms of a are as follows:


9q


9q + 1


9q + 2


9q + 3


9q + 4


9q + 5


9q + 6


9q + 7


9q + 8


Now to find the cubes of these values we have the following expansion formula:


(a + b)3 = a3 + 3a2b + 3ab2 + b3


(9q + b)3 = 729q3 + 243q2b + 27qb2 + b3


Now when we divide the above equation by 9 we get the quotient as


81q3 + 27q2b + 3qb2 and the remainder is b3.


So we have to consider the value of b3


So to put b = 0


We get 9m + 0 = 9m


So to put b = 1


We get 13 = 1 so we get = 9m + 1


So to put b = 2


We get 23 = 8 so we get = 9m + 8


So to put b = 3


We get 33 = 27 and it is divisible by 9, so we get = 9m


So to put b = 4


We get 43 = 64 and when divided by 9 we get 1 as remainder, so we get = 9m + 8


So to put b = 5


We get 53 = 125 and when divided by 9 we get 8 as remainder, so we get = 9m + 8


So to put b = 6


We get 63 = 216 and when divided by 9 we get 0 as remainder, so we get = 9m


So to put b = 7


We get 73 = 343 and when divided by 9 we get 1 as remainder, so we get = 9m + 1


So to put b = 8


We get 83 = 512 and when divided by 9 we get 8 as remainder, so we get = 9m + 8


So it is proved that all the values are in the form of 9m, 9m + 1 and 9m + 8.



Question 3.

Show that any positive odd integral number can be expressed as 6q + 1 or 6q + 3 or 6q + 5 where q is a positive integer.


Answer:

Let take a as any positive integer and b = 6.
Then using Euclid’s algorithm we get a = 6q + r here r is remainder and value of q is more
than or equal to 0 and r = 0, 1, 2, 3, 4, 5 because 0 ≤r < b and the value of b is 6

So total possible forms will 6q + 0 , 6q + 1 , 6q + 2,6q + 3,6q + 4,6q + 5
6q + 0, 6 is divisible by 2 so it is an even number



6q + 1, 6 is divisible by 2 but 1 is not divisible by 2 so it is an odd number



6q + 2, 6 is divisible by 2 and 2 is also divisible by 2 so it is an even number



6q + 3, 6 is divisible by 2 but 3 is not divisible by 2 so it is an odd number



6q + 4, 6 is divisible by 2 and 4 is also divisible by 2 it is an even number



6q + 5, 6 is divisible by 2 but 5 is not divisible by 2 so it is an odd number



So odd numbers will in form of 6q + 1, or 6q + 3, or 6q + 5



Question 4.

Find the HCF (Highest Common Factor) of the following pairs of number by Euclid Division Algorithm:

210, 55


Answer:

From the integers given in the question 210 and 55, it is observed that 210 > 55. So by Euclid’s Division Lemma we get the following:

210 = 55 × 3 + 45


Her the remainder is 45 which is not equal to zero. So applying Euclid’s Division Lemma on divisor 55 and remainder 45.


55 = 45 × 1 + 10


Here the remainder is 10 which is not equal to zero. So applying Euclid’s Division Lemma on divisor 45 and remainder 10.


45 = 10 × 4 + 5


Here the remainder is 5 which is not equal to zero. So applying Euclid’s Division Lemma on divisor 10 and remainder 5.


10 = 5 × 2 + 0


So from the above relation is seen that remainder zero is obtained.


So the HCF of 210 and 55 is 5.


The entire process can be expressed in the following way:




Question 5.

Find the HCF (Highest Common Factor) of the following pairs of number by Euclid Division Algorithm:

420, 130


Answer:

From the integers given in the question 420 and 130, it is observed that 420 > 130. So by Euclid’s Division Lemma we get the following:

420 = 130 × 3 + 30


Here the remainder is 30 which is not equal to zero. So applying Euclid’s Division Lemma on divisor 130 and remainder 30.


130 = 30 × 4 + 10


Her the remainder is 10 which is not equal to zero. So applying Euclid’s Division Lemma on divisor 30 and remainder 10.


30 = 10 × 3 + 0


So from the above relation is seen that remainder zero is obtained.


So the HCF of 420 and 130 is 10.


The entire process can be expressed in the following way:




Question 6.

Find the HCF (Highest Common Factor) of the following pairs of number by Euclid Division Algorithm:

75, 243


Answer:

From the integers given in the question 243 and 75, it is observed that 243 > 75. So by Euclid’s Division Lemma we get the following:

243 = 75 × 3 + 18


Here the remainder is 18 which is not equal to zero. So applying Euclid’s Division Lemma on divisor 75 and remainder 18.


75 = 18 × 4 + 3


Here the remainder is 3 which is not equal to zero. So applying Euclid’s Division Lemma on divisor 18 and remainder 3.


18 = 3 × 6 + 0


So from the above relation is seen that remainder zero is obtained.


So the HCF of 243 and 75 is 3.


The entire process can be expressed in the following way:




Question 7.

Find the HCF (Highest Common Factor) of the following pairs of number by Euclid Division Algorithm:

135, 225


Answer:

From the integers given in the question 225 and 135, it is observed that 225 > 135. So by Euclid’s Division Lemma we get the following:

225 = 135 × 1 + 90


Here the remainder is 90 which is not equal to zero. So applying Euclid’s Division Lemma on divisor 135 and remainder 90.


135 = 90 × 1 + 45


Here the remainder is 45 which is not equal to zero. So applying Euclid’s Division Lemma on divisor 90 and remainder 45.


90 = 45 × 2 + 0


So from the above relation is seen that remainder zero is obtained.


So the HCF of 135 and 225 is 45.


The entire process can be expressed in the following way:




Question 8.

Find the HCF (Highest Common Factor) of the following pairs of number by Euclid Division Algorithm:

196, 38220


Answer:

From the integers given in the question 38220 and 196, it is observed that 38220 > 196. So by Euclid’s Division Lemma we get the following:

38220 = 196 × 195 + 0


So from the above relation is seen that remainder zero is obtained.


So the HCF of 38220 and 196 is 195.


The entire process can be expressed in the following way:




Question 9.

Find the HCF (Highest Common Factor) of the following pairs of number by Euclid Division Algorithm:

867, 255


Answer:

From the integers given in the question 867 and 255, it is observed that 867 > 255. So by Euclid’s Division Lemma we get the following:

867 = 255 × 3 + 102


Here the remainder is 102 which is not equal to zero. So applying Euclid’s Division Lemma on divisor 255 and remainder 102.


255 = 102 × 2 + 51


Here the remainder is 51 which is not equal to zero. So applying Euclid’s Division Lemma on divisor 102 and remainder 51.


102 = 51 × 2 + 0


So from the above relation is seen that remainder zero is obtained.


So the HCF of 867 and 255 is 51.


The entire process can be expressed in the following way:




Question 10.

If the highest common factor of numbers 408 and 1032 is expressed in the form of 1032x – 408 × 5, then find the value of x.


Answer:

If the HCF of 408 and 1032 can be written as

By Euclid’s division algorithm,


1032 = 408×2 + 216


408 = 216×1 + 192


216 = 192×1 + 24


192 = 24×8 + 0


Since the remainder becomes 0 here, so HCF of 408 and 1032 is 24


Now,


1032x - 408*5 = HCF of these numbers


⟹ 1032x - 2040 = 24


⟹ 1032x = 24 + 2040


⟹ x = 2064/1032


⟹ x = 2


So value of x is 2.




Exercise 2.2
Question 1.

Express the following numbers as a product of its prime factors:

468


Answer:

The number 468 can be expressed as follows:

468 = 2 × 2 × 3 × 3 × 13




Question 2.

Express the following numbers as a product of its prime factors:

945


Answer:

The number 945 can be expressed as follows:

945 = 5 × 7 × 3 × 3 × 3




Question 3.

Express the following numbers as a product of its prime factors:

140


Answer:

The number 140 can be expressed as follows:

140 = 2 × 2 × 5 × 7




Question 4.

Express the following numbers as a product of its prime factors:

3825


Answer:

The number 3825 can be expressed as follows:

3825 = 17 × 5 × 5 × 3 × 3



Question 5.

Express the following numbers as a product of its prime factors:

20570


Answer:

The number 20570 can be expressed as follows:

20570 = 17 × 5 × 2 × 11 × 11




Question 6.

Find the HCF and LCM of the following pairs of integers and verify that HCF × LCM = Product of integers.

96 and 404


Answer:

The numbers are expressed as follows:

96 = 2 × 2 × 2 × 2 × 2 × 3


404 = 2 × 2 × 101


So for finding the HCF we may write as follows:



So HCF = 4


For finding the LCM we may write as follows:



So LCM = 25 × 3 × 101


= 9696


So Product of HCM × LCM = 9696 × 4


= 38784


So product of numbers = 404 × 96


= 38784



Question 7.

Find the HCF and LCM of the following pairs of integers and verify that HCF × LCM = Product of integers.

336 and 54


Answer:

The numbers are expressed as follows:

336 = 2 × 2 × 2 × 2 × 3 × 7


54 = 2 × 3 × 3 × 3


So for finding the HCF we may write as follows:



So HCF = 2 × 3


= 6


For finding the LCM we may write as follows:



So LCM = 24 × 33 × 7


= 3024


So Product of HCM × LCM = 3024 × 6


= 18144


So product of numbers = 54 × 336


= 18144



Question 8.

Find the HCF and LCM of the following pairs of integers and verify that HCF × LCM = Product of integers.

90 and 144


Answer:

The numbers are expressed as follows:

336 = 2 × 5 × 3 × 3


144 = 2 × 2 × 2 × 2 × 3 × 3


So for finding the HCF we may write as follows:



So HCF = 2 × 32


= 18


For finding the LCM we may write as follows:



So LCM = 24 × 32 × 5


= 720


So Product of HCM × LCM = 720 × 18


= 12960


So product of numbers = 90 × 144


= 12960



Question 9.

Find the HCF and LCM of the following integers by applying the prime factorization method.

12, 15 and 21


Answer:

The numbers are expressed as follows:

12 = 2 × 2 × 3


15 = 3 × 5


21 = 7 × 3


So for finding the HCF we may write as follows:



So HCF = 3


= 3


For finding the LCM we may write as follows:



So LCM = 22 × 3 × 5 × 7


= 420



Question 10.

Find the HCF and LCM of the following integers by applying the prime factorization method.

24, 15 and 36


Answer:

The numbers are expressed as follows:

24 = 2 × 2 × 2 × 3


15 = 3 × 5


36 = 2 × 2 × 3 × 3


So for finding the HCF we may write as follows:



So HCF = 3


For finding the LCM we may write as follows:



So LCM = 23 × 32 × 5


= 360



Question 11.

Find the HCF and LCM of the following integers by applying the prime factorization method.

17, 23 and 29


Answer:

The numbers can be expressed as follows:

17 = 17 × 1


23 = 23 × 1


29 = 29 × 1


So HCF = 1


For finding the LCM we may write as follows:



So LCM = 17 × 23 × 29 × 1


= 11339



Question 12.

Find the HCF and LCM of the following integers by applying the prime factorization method.

6, 72 and 120


Answer:

The numbers can be expressed as follows:

6 = 2 × 3


72 = 2 × 2 × 2 × 3 × 3


120 = 2 × 2 × 2 × 3 × 5


So for finding the HCF we may write as follows:



So HCF = 2 × 3


= 6


For finding the LCM we may write as follows:



So LCM = 23 × 32 × 5


= 360



Question 13.

Find the HCF and LCM of the following integers by applying the prime factorization method.

40, 36 and 126


Answer:

The numbers can be expressed as follows:

40 = 2 × 2 × 2 × 5


36 = 2 × 2 × 3 × 3


126 = 2 × 3 × 3 × 7


So for finding the HCF we may write as follows:



So HCF = 2


For finding the LCM we may write as follows:



So LCM = 23 × 32 × 5 × 7


= 2520



Question 14.

Find the HCF and LCM of the following integers by applying the prime factorization method.

8, 9 and 25


Answer:

The numbers can be expressed as follows:

8 = 2 × 2 × 2 × 1


9 = 3 × 3 × 1


25 = 5 × 5 × 1


So for finding the HCF we may write as follows:



So HCF = 1


For finding the LCM we may write as follows:



So LCM = 23 × 32 × 52 × 1


= 1800



Question 15.

Raman takes 18 minutes to drive one round of the field on circular path of a playground, while Anupriya takes 12 minutes to drive one round of the field on this circular path. Suppose they both start at the same point and at the same time, and go in the same direction. After how much time will they meet again at the starting point?


Answer:

In the question it is mentioned that Anupriya takes 12 minutes and Raman takes 18 minutes. Since Anupriya takes lesser time as compared to Raman, so they both will meet again at the same time when Anupriya completes one round with respect to Raman. So the total time taken will be the LCM of 12 and 18 minutes.

12 = 2 × 2 × 3


18 = 2 × 3 × 3S


So LCM of 12 and 18 can be calculated as follows:



So LCM = 22 × 32


= 36


So they both will meet after 36 minutes.



Question 16.

In a seminar, the number of participants in Hindi, English and Mathematics are 60, 84 and 108 respectively. If in each room the same number of participants of the same subject are seated then find the minimum number of rooms required.


Answer:

Given:

Number of participants for Hindi = 60


Number of participants for English = 84


Number of participants for Mathematics = 108


Number of participants for each room will be the HCF of 60, 84 and 108.


60 = 2 × 2 × 3 × 5


84 = 2 × 2 × 3 × 7


108 = 2 × 2 × 3 × 3 × 3


So for finding the HCF we may write as follows:



So HCF = 22 × 3


= 12


So total number of participants = 60 + 54 + 108


= 252


Number of rooms required = 252 / 12


= 21


So total 21 rooms are required.




Exercise 2.3
Question 1.

Prove that is an irrational number.


Answer:

Let us consider that 5 - √3 is a rational number.

Let 5 - √3 = a/b for b ≠ 0 ……………… (i)


Where a and b are co-prime integer numbers.


From (i) we can write as follows:


5 – a/b = √3



Now from the LHS we know that a and b are integral numbers and therefore the fraction will also be rational number which implies that √3 has to be a rational number. But we know that √3 is an irrational number.


So our assumption at the beginning of the problem is proved false.


So 5 - √3 is an irrational number.



Question 2.

Prove that the following numbers are irrational numbers:




Answer:

Let us consider that 1/√2 is a rational number.

Let 1/√2 = a/b for b ≠ 0 ……………… (i)


Where a and b are co-prime integer numbers.


From (i) we can write as follows:


b = a√2


b2 = 2a2 …………………………….. (i)


Now since 2a2 is divisible by 2, so b2 has to be divisible by 2. From theorem 2.3 we can clearly states that if b2 is divided by 2 so b is also divisible by 2. So we conclude that 2 divides b.


Now we can write the integer b in following format,


b = 2c


b2 = 4c2 …………………………… (ii)


By comparing (i) and (ii) we can state as follows:


4c2 = 2a2


a2 = 2c2


From the above equation we can conclude that a2 is divisible by 2 and also by a.


From the values of a and b it is seen that a and b has a common factor and it is clearly indicates that 2 is a common factor. But it is assumed in the beginning that a and b has no common factors.


So our assumption made at the beginning of the problem is wrong.


Hence it is proved that 1/√2 is an irrational number.



Question 3.

Prove that the following numbers are irrational numbers:




Answer:

Let us consider that 6 + √2 is a rational number.

Let 6 + √2 = a/b for b ≠ 0 ……………… (i)


Where a and b are co-prime integer numbers.


From (i) we can write as follows:


a/b - 6 = √2



Now from the LHS we know that a and b are integral numbers and therefore the fraction will also be rational number which implies that √2 has to be a rational number. But we know that √2 is an irrational number.


So our assumption at the beginning of the problem is proved false.


So 6 + √2 is an irrational number.



Question 4.

Prove that the following numbers are irrational numbers:




Answer:

Let us consider that 1/√2 is a rational number.

Let 3√2 = a/b for b ≠ 0 ……………… (i)


Where a and b are co-prime integer numbers.


From (i) we can write as follows:


a = 3b√2


a2 = 18b2 …………………………….. (i)


Now since 18b2 is divisible by 18, so a2 has to be divisible by 18. From theorem 2.3 we can clearly states that if a2 is divided by 18 so a is also divisible by 18. So we conclude that a divides 18.


Now we can write the integer b in following format,


a = 18c


a2 = 324c2 …………………………… (ii)


By comparing (i) and (ii) we can state as follows:


324c2 = 18b2


b2 = 18c2


From the above equation we can conclude that b2 is divisible by 18 and also by 18.


From the values of a and b it is seen that a and b has a common factor and it is clearly indicates that 18 is a common factor. But it is assumed in the beginning that a and b has no common factors.


So our assumption made at the beginning of the problem is wrong.


Hence it is proved that 3√2 is an irrational number.



Question 5.

If p and q are co-prime positive integers then prove that is an irrational number.


Answer:

Let √p + √q be a rational number.

Let √p + √q = x where x is integral number.


Squaring both sides.


(√p + √q)2 = x2


p + q + 2√pq = x2



Since p and q are co-prime positive integers. So root of p and root of q will definitely be an irrational numbers as they are not perfect squares. So √pq has to be an irrational number.


So the assumption made at the beginning of the problem is false.


So it is proved that √p + √q is an irrational number.




Exercise 2.4
Question 1.

Without actually performing the long division, state whether the following rational numbers will have a terminating decimal expansion or a non-terminating repeating decimal expansion:

(i)

(ii)

(iii)

(iv)

(v)

(vi)

(vii)


Answer:

(i)






Hence we can see that the denominator is of the form 2m × 5n and hence decimal expansion of 15/1600 is of terminating type.


(ii)






Hence we can see that the denominator is of the form 2m × 5n and hence decimal expansion of 13/3125 is of terminating type.


(iii)


Hence we can see that the denominator is of the form 2m × 5n and hence decimal expansion of 23/200 is of terminating type.


(iv)


Hence we can see that the denominator is not of the form 2m × 5n and hence decimal expansion of 17/6 is of non-terminating type.


(v)


Hence we can see that the denominator is not of the form 2m × 5n and hence decimal expansion of 129 / (22 × 57 × 75) is of non-terminating type.


(vi)




Hence we can see that the denominator is of the form 2m × 5n and hence decimal expansion of 35/50 is of terminating type.


(vii)




Hence we can see that the denominator is of the form 2m × 5n and hence decimal expansion of 7/80 is of terminating type.



Question 2.

Write the decimal expansions of the following rational numbers and decide whether they are terminating or not.

(i)

(ii)

(iii)


Answer:

(i)



= 0.104


Hence we can see that the denominator is of the form 2m × 5n and hence decimal expansion of 13/125 is of terminating type.


(ii)




= 23.3408


Hence we can see that the denominator is of the form 2m × 5n and hence decimal expansion of 14588/625 is of terminating type.


(iii)




= 0.098


Hence we can see that the denominator is of the form 2m × 5n and hence decimal expansion of 49/500 is of terminating type.



Question 3.

For the decimal expansions given below decide whether this number is a rational number or not. If it is a rational number then write your note about the prime factorization of its denominator.

(i) 0.120120012000120000….

(ii) 43.123456789

(iii)


Answer:

(i) Since the given decimal expansion has non-terminating expansion which implies that it is an irrational number.

(ii) We know the general form of a rational number is p/q



Now let us find out the prime factorization of the denominator.




Hence the denominator is in the form of 2m × 5n.


(iii) Since the given decimal expansion has non-terminating expansion which implies that it is an irrational number.




Miscellaneous Exercise 2
Question 1.

The sum of the powers of the prime factors of 196 is:
A. 1

B. 2

C. 4

D. 6


Answer:

The Square of 14 is 196.

196 = 14 × 14


= 2 × 2 × 7 × 2


= 22 × 72


Power of 2 = 2


Power of 7 = 2


Sum of powers = 2 + 2


= 4


So correct answer is C.


Question 2.

If two numbers are written in the form of m = pq3 and n = p3q2 then find the HCF of m, n when p, q are prime numbers:
A. pq

B. pq2

C. p2q2

D. p3q3


Answer:

m = pq3

n = p3q2


So for finding the HCF we may write as follows:


Common Factor Least Power


p 1


q 2


So HCF = q2 × p


= pq2


So correct answer is B.


Question 3.

The HCF (Highest Common Factor) of 95 and 152 is :
A. 1

B. 19

C. 57

D. 38


Answer:

The numbers can be expressed as follows:

95 = 5 × 19


152 = 2 × 2 × 2 × 19


So for finding the HCF we may write as follows:


Common Factor Least Power


19 1


So HCF = 19


So correct answer is B.


Question 4.

The product of two numbers is 1080. Their HCF is 30. Then their LCM is:
A. 5

B. 16

C. 36

D. 108


Answer:

We know the basic following relation:

HCF × LCM = 1st Number × 2nd Number


30 × LCM = 1080


LCM = 1080 / 30


LCM = 36


So correct answer is C.


Question 5.

The decimal expansion of the number will be:
A. terminating

B. non-terminating repeating

C. terminating and non-terminating both

D. the number is not a rational number.


Answer:

From the question we can see that the denominator is not of the form 2m × 5n and hence decimal expansion of 441 / (22 × 57 × 72) is of non-terminating repeating type.

So correct answer is B.


Question 6.

After how many decimal places with the rational number terminate?
A. one

B. two

C. three

D. four


Answer:





⟹ 0.086


So it is clearly seen that after three decimal places the rational number is terminating


So the correct answer is C.


Question 7.

The least number by which multiplying to a rational number is obtained will be:
A. 3

B.

C. 9

D.


Answer:

When 3 will be multiplied with √27 it would be 3√27 which is equal to 9√3 which is not rational number.

When √3 will be multiplied with √27 then it would result in √81 which is 9 and a rational number.


When 9 will be multiplied with √27 it would be 9√27 which is equal to 27√3 which is not rational number.


When 3√3 will be multiplied with √27 then it would result in 3√81 which is 27 and a rational number.


Since the number to be multiplied has to be the smallest the correct answer is √3.


So correct answer is B.


Question 8.

If for two rational numbers HCF = LCM, then the number must use:
A. composite

B. equal

C. prime

D. co-prime


Answer:

If the LCM = HCF, then the two numbers has to be equal.

So the correct answer is B.


Question 9.

If LCM of a and 18 is 36 and HCF of a and 18 is 2, then the value of a will be:
A. 1

B. 2

C. 5

D. 4


Answer:

We know the basic following relation:

HCF × LCM = 1st Number × 2nd Number


1st Number = a


2nd Number = 18


36 × 2 = 18 × a


a = 72 / 18


a = 4


So the correct answer is D.


Question 10.

If n is a rational number, then units digit in 6n – 5n is:
A. 1

B. 6

C. 5

D. 9


Answer:

Any power of 6 will have 6 at its units place and any power of 5 will have 5 at its units place. So when we subtract the units digits that is 6 – 5 we get 1 as the answer.

So the correct answer is A.


Question 11.

If is a rational number, then what will be the condition on q when is a terminating decimal?


Answer:

For the decimal to be of terminating nature q has to be of the form 2m × 5n.



Question 12.

Simplifying state whether the number is a rational number or an irrational number?


Answer:





⟹ 6


So the answer is a rational number.



Question 13.

Show that any positive odd integer is of the form 4q + 1 or 4q + 3. Where q is some integer.


Answer:

Let a be any odd integer and b = 4. So by applying Euclid’s Division Lemma,

So we have a = 4q + r


Where r is in the range of 0 to 4.


Case I:


For r = 0


a = 4q


Case II:


For r = 1


a = 4q + 1


Case III:


For r = 2


a = 4q + 2


a = 2 (2q + 1)


Case IV:


For r = 3


a = 4q + 3


Since 4q and 4q + 2 is multiple of 2. Thus they are even numbers.


Hence any positive odd integers is in the form of 4q + 1 and 4q + 3.


Hence Proved.



Question 14.

Prove that the product of two executive positive integers is divisible by 2.


Answer:

Let n and n-1 be the 2 positive integers.

Product = n × (n-1)


= n2 – n


Case I (when n is even):


Let n = 2q


n2 – n = (2q) 2 - 2q


= 4q2 - 2q


= 2q × (2q-1)


Hence the product n2 – n is divisible by 2


Case II (when n is odd):


Let n be 2q + 1


n2 – n = (2q + 1)2- (2q + 1)


= 4q2 + 4q + 1 - 2q – 1


= 4q2 + 2q


= 2q × (2q + 1)


Hence the product n2 – n is divisible by 2


Hence Proved



Question 15.

Find the greatest number by which on dividing 2056 and 967 the remainder are obtained as 5 and 7 respectively.


Answer:

It is mentioned in the question that that on dividing 2053 by the required number there is a remainder of 5 which means that 2053 - 5 = 2048 is exactly divisible by required number . In the similar manner,


967 - 7 = 960 the required number is the largest number satisfying the above property.


Therefore, it is the HCF of 2048 & 960


2048 = 64 × 32


= 26 × 25


= 211


960 = 5 × 2 × 2 × 2 × 2 × 2 × 2 × 3


So for finding the HCF we may write as follows:


Common Factor Least Power


2 6


So HCF = 26


= 64


HCF of 2048 & 960 is 64


Hence required number is 64



Question 16.

Explain why 7 × 11 × 13 + 13 and 7 × 6 × 5 × 4 × 3 × 2 × 1 + 5 are composite numbers?


Answer:

By definition,

A composite number is a positive integer that has a factor other than 1 and itself. Now considering your numbers,


7×11×13 + 13 may be written as, i.e. 13 * (78). So other than 1 and the number itself, 13 and 78 are also the factors of the number. Further, 78 = 39 x 2. So, 39 and 2 are also its factors. So this number is definitely not prime. Hence its composite number.


Similarly, 7×6×5×4×3×2×1 + 5 can be written as , i.e. 5 * (1009). So, other than the number and 1, it have 5 and 1009 as its factors too. So it is also a composite number.



Question 17.

If the HCF of two numbers 306 and 657 is 9 then find their LCM.


Answer:

We know the basic relation:

HCF × LCM = 1st Number × 2nd Number


1st Number = 306


2nd Number = 657


HCF = 9


LCM = (306 × 657) / 9


= 34 × 657 [as 306 / 9 = 34]


= 22,338


So LCM = 22338



Question 18.

A rectangular verandah is 18 m 72 cm long and 13 m 20 cm wide. Square tiles of the same dimensions are to paired in it. Find the least number of such tiles.


Answer:

Length of rectangular verandah = 1872 cm.

Breadth of courtyard = 1320 cm


HCF of the above measurements:


1872 = 1320 × 1 + 552


1320 = 552 × 2 + 216


552 = 216 × 2 + 120


216 = 120 × 1 + 96


120 = 96 × 1 + 24


96 = 24 × 4 + 0


HCF of 1872 and 1320 = 24 cm


Number of tiles = Area of courtyard / areas of side of tiles


= (1872 × 1320) / (24 × 24)


= 4290


Hence total number of tiles required is 4290.



Question 19.

Prove that the following numbers are irrational numbers:




Answer:

Let us consider that 5 - √3 is a rational number.

Let 5√2 = a/b for b ≠ 0 ……………… (i)


Where a and b are co-prime integer numbers.


From (i) we can write as follows:


a/5b = √2


Now from the LHS we know that a and b are integral numbers and therefore the fraction will also be rational number which implies that √2 has to be a rational number. But we know that √2 is an irrational number.


So our assumption at the beginning of the problem is proved false.


So 5√2 is an irrational number.



Question 20.

Prove that the following numbers are irrational numbers:




Answer:

Let us consider that 2/√7 is a rational number.

Let 2/√7 = a/b for b ≠ 0 ……………… (i)


Where a and b are co-prime integer numbers.


From (i) we can write as follows:


2b/a = √7


Now from the LHS we know that a and b are integral numbers and therefore the fraction will also be rational number which implies that √7 has to be a rational number. But we know that √7 is an irrational number.


So our assumption at the beginning of the problem is proved false.


So 2/√7 is an irrational number.



Question 21.

Prove that the following numbers are irrational numbers:




Answer:

Let us consider that 3/2√5 is a rational number.

Let 3/2√5 = a/b for b ≠ 0 ……………… (i)


Where a and b are co-prime integer numbers.


From (i) we can write as follows:


3b/2a = √5


Now from the LHS we know that a and b are integral numbers and therefore the fraction will also be rational number which implies that √5 has to be a rational number. But we know that √5 is an irrational number.


So our assumption at the beginning of the problem is proved false.


So 3/2√5 is an irrational number.



Question 22.

Prove that the following numbers are irrational numbers:




Answer:

us consider that 2 + √2 is a rational number.

Let 2 + √2 = a/b for b ≠ 0 ……………… (i)


Where a and b are co-prime integer numbers.


From (i) we can write as follows:


a/b – 2 = √2



Now from the LHS we know that a and b are integral numbers and therefore the fraction will also be rational number which implies that √2 has to be a rational number. But we know that √2 is an irrational number.


So our assumption at the beginning of the problem is proved false.


So 2 + √2 is an irrational number.



Question 23.

What can you say about the prime factorization of the denominators of the following rational number?

(i) 34.12345

(ii)


Answer:

(i) We know the general form of a rational number is p/q


Now let us find out the prime factorization of the denominator.





Hence the denominator is in the form of 2m × 5n.


(ii) Since the above given number has non-terminating decimal expansion, the given number is an irrational number. So it is not possible to find the prime factorization of the given number.